Frage:
Ableitung der Maxwellschen Gleichungen aus dem Feldtensor Lagrangian
amc
2011-01-16 14:18:20 UTC
view on stackexchange narkive permalink

Ich habe in meiner Freizeit angefangen, Peskin und Schroeder zu lesen, und bin etwas verwirrt darüber, wie man Maxwells Gleichungen aus der (quellenfreien) Lagrange-Dichte $ L = - \ frac {1} {4} erhält. F _ {\ mu \ nu} F ^ {\ mu \ nu} $ (wobei $ F ^ {\ mu \ nu} = \ partiell ^ \ mu A ^ \ nu - \ partiell ^ \ nu A ^ \ mu $ das ist Feldtensor).

Einsetzen der Definition des Feldtensors ergibt $ L = - \ frac {1} {2} [(\ teilweise_ \ mu A_ \ nu) (\ teilweise ^ \ mu A. ^ \ nu) - (\ teilweise_ \ mu A_ \ nu) (\ teilweise ^ \ nu A ^ \ mu)] $. Ich weiß, ich sollte $ A ^ \ mu $ als dynamische Variable in den Euler-Lagrange-Gleichungen verwenden, die zu $ ​​\ frac {\ partiell L} {\ partiell A_ \ mu} - \ partiell_ \ mu \ frac {\ partiell werden L} {\ partiell (\ partiell_ \ mu A_ \ nu)} = - \ partiell_ \ mu \ frac {\ partiell L} {\ partiell (\ partiell_ \ mu A_ \ nu)} $, aber ich bin verwirrt darüber, wie um von hier fortzufahren.

Ich weiß, ich sollte am Ende $ \ partielle_ \ mu F ^ {\ mu \ nu} = 0 $ haben, aber ich verstehe nicht ganz warum. Da $ \ mu $ und $ \ nu $ Dummy-Indizes sind, sollte ich sie ändern können: In welcher Beziehung stehen die Indizes im Lagrange zu den Indizes in den Ableitungen in den Euler-Lagrange-Gleichungen?

Siehe in Sean Carrolls Buch.Voller Ableitung dort
Warum reicht es nicht aus, die $ F ^ {\ mu \ nu} = \ partielle {[\ mu} A {\ nu]} $ in Maxwells Gleichungen einzufügen und zu zeigen, dass sie gelten?
Fünf antworten:
Pablo
2011-09-19 18:36:46 UTC
view on stackexchange narkive permalink

Wir variieren die Aktion $$ \ delta \ int {L \; \ mathrm {d} t} = \ delta \ int {\ int {\ Lambda \ left ({A_ \ nu, \ teilweise _ \ mu A_ \ nu} \ right) \ mathrm {d} ^ 3 x \; \ mathrm {d} t = 0}} $$$ {\ Lambda \ left ({A_ \ nu, \ teilweise _ \ mu A_ \ nu} \ rechts )} $ ist die Lagrange-Dichte des Systems.

Also, $$ \ int {\ int {\ left ({\ frac {{\ partielle \ Lambda}} {{\ partielle A_ \ nu }} \ Delta A_ \ nu + \ frac {{\ partiell \ Lambda}} {{\ partiell \ links ({\ partiell _ \ mu A_ \ nu} \ rechts)}} \ Delta \ links ({\ partiell _ \ mu A_ \ nu} \ right)} \ right) \ mathrm {d} ^ 3 x \; \ mathrm {d} t = 0}} $$ Durch Integration nach Teilen erhalten wir: $$ \ int {\ int {\ links ({\ frac {{\ partiell \ Lambda}} {{\ partiell A_ \ nu}} - \ partiell _ \ mu \ frac {{\ partiell \ Lambda}} {{\ partiell \ links ({\ partiell _ \ mu A_ \ nu} \ rechts)}} \ rechts) \ delta A_ \ nu \ mathrm {d} ^ 3 x \; \ mathrm {d} t = 0}} \ impliziert \ frac {{\ partiell \ Lambda} } {{\ partielle A_ \ nu}} - \ partielle _ \ mu \ frac {{\ partielle \ Lambda}} {{\ partielle \ links ({\ partielle _ \ mu A_ \ nu} \ rechts)}} = 0 $$ Wir müssen die Dichte des Lagrange bestimmen. Ein Begriff befasst sich mit der Wechselwirkung der Ladungen mit dem elektromagnetischen Feld $ J ^ \ mu A_ \ mu $. Der andere Begriff ist die Energiedichte des elektromagnetischen Feldes: Dieser Begriff ist die Differenz des Magnetfelds und des elektrischen Feldes. Wir haben also: $$ \ Lambda = J ^ \ mu A_ \ mu + \ frac {1} {{4 \ mu _0}} F ^ {\ mu \ nu} F _ {\ mu \ nu} $$ Wir haben: $$ \ frac {{\ partielle \ Lambda}} {{\ partielle A_ \ nu}} = J ^ \ nu $$ also: \ begin {align} \ teilweise _ \ mu \ frac {{\ teilweise \ Lambda}} {{\ teilweise \ links ({\ teilweise _ \ mu A_ \ nu} \ rechts)}} & = \ frac {1} { {4 \ mu _0}} \ teilweise _ \ mu \ links ({\ frac {\ teilweise} {{\ teilweise \ links ({\ teilweise _ \ mu A_ \ nu} \ rechts)}} F ^ {\ kappa \ lambda} F _ {\ kappa \ lambda}} \ rechts) \\ & = \ frac {1} {{4 \ mu _0}} \ teilweise _ \ mu \ links ({\ frac {\ teilweise} {{\ teilweise \ links ({\ partielle _ \ mu A_ \ nu} \ rechts)}} \ links ({\ links ({\ partielle ^ \ kappa A ^ \ lambda - \ partielle ^ \ lambda A ^ \ kappa} \ rechts) \ links ({\ partiell _ \ kappa A_ \ lambda - \ partiell _ \ lambda A_ \ kappa} \ rechts)} \ rechts)} \ rechts) \\ & = \ frac {1} {{4 \ mu _0}} \ partiell _ \ mu \ left ({\ frac {\ partiell} {{\ partiell \ links ({\ partiell _ \ mu A_ \ nu} \ rechts)}} \ links ({\ partiell ^ \ kappa A ^ \ lambda \ partiell) _ \ kappa A_ \ lambda - \ partiell ^ \ kappa A ^ \ lambda \ partiell _ \ lambda A_ \ kappa - \ partiell ^ \ lambda A ^ \ kappa \ partiell _ \ kappa A_ \ lambda + \ partiell ^ \ lambda A ^ \ kappa \ partielle _ \ lambda A_ \ kappa} \ rechts)} \ rechts) \ end {align} Der dritte und der vierte sind die gleichen wie der erste und der zweite Term. Sie können $ k \ leftrightarrow \ lambda $ ausführen: \ begin {align} \ partielle _ \ mu \ frac {{\ partielle \ Lambda}} {{\ partielle \ linke ({\ partielle _ \ mu A_ \ nu} \ rechts )}} & = \ frac {1} {{2 \ mu _0}} \ teilweise _ \ mu \ links ({\ frac {\ teilweise} {{\ teilweise \ links ({\ teilweise _ \ mu A_ \ nu}) \ rechts)}} \ links ({\ partiell ^ \ kappa A ^ \ lambda \ partiell _ \ kappa A_ \ lambda - \ partiell ^ \ kappa A ^ \ lambda \ partiell _ \ lambda A_ \ kappa} \ rechts)} \ rechts) \ ;. \ end {align} Aber \ begin {align} \ frac {\ partiell} {{\ partiell \ links ({\ partiell _ \ mu A_ \ nu} \ rechts)}} \ links ({\ partiell ^ \ kappa A ^ \ lambda \ partiell _ \ kappa A_ \ lambda} \ rechts) & = \ partiell ^ \ kappa A ^ \ lambda \ frac {\ partiell} {{\ partiell \ links ({\ partiell _ \ mu A_ \ nu} \ rechts)}} \ links ( {\ partiell _ \ kappa A_ \ lambda} \ rechts) + \ partiell _ \ kappa A_ \ lambda \ frac {\ partiell} {{\ partiell \ links ({\ partiell _ \ mu A_ \ nu} \ rechts)}} \ left ({\ partiell ^ \ kappa A ^ \ lambda} \ rechts) \\ & = \ partiell ^ \ kappa A ^ \ lambda \ delta _ \ kappa ^ \ mu \ delta _ \ lambda ^ \ nu + g ^ { \ kappa \ alpha} g ^ {\ lambda \ beta} \ partiell _ \ kappa A_ \ lambda \ frac {\ partiell} {{\ partiell \ links ({\ partiell _ \ mu A_ \ nu} \ rechts)}} \ links ({\ partiell _ \ alpha A_ \ beta} \ rechts) \\ & = 2 \ partiell ^ \ mu A ^ \ nu \ ;. \ end {align}

Wir haben:

\ begin {align} \ frac {\ partiell} {{\ partiell \ links ({\ partiell _ \ mu A_ \ nu} \ rechts)}} \ links ({\ partiell ^ \ kappa A ^ \ lambda \ partiell _ \ lambda A_ \ kappa} \ rechts) & = 2 \ partiell ^ \ nu A. ^ \ mu \;. \ end {align}

Also,

\ begin {align} \ teilweise _ \ mu \ left ({\ frac {{\ teilweise \ Lambda}} {{\ teilweise \ links ({\ partiell _ \ mu A_ \ nu} \ rechts)}}} \ rechts) & = \ frac {1} {{\ mu _0}} \ partiell _ \ mu \ links ({\ partiell ^ \ mu A. ^ \ nu - \ partiell ^ \ nu A ^ \ mu} \ rechts) \\ & = \ frac {1} {{\ mu _0}} \ partiell _ \ mu F ^ {\ mu \ nu} \ ;. \ end {align} Die Lagrange-Gleichungen liefern die nicht homogenen Maxwell-Gleichungen:

$$ \ partielle _ \ mu F ^ {\ mu \ nu} = \ mu _0 J ^ \ nu \;. $$

Zu Ihrer Information, diese Antwort (v4) verwendet implizit die Vorzeichenkonvention $ (+, -, -, -) $.
Marek
2011-01-16 14:46:19 UTC
view on stackexchange narkive permalink

Nun, Sie sind fast da. Verwenden Sie die Tatsache, dass $$ {\ partiell (\ partiell _ {\ mu} A _ {\ nu}) \ über \ partiell (\ partiell _ {\ rho} A _ {\ sigma})} = \ delta _ {\ mu} ^ {\ rho} \ delta _ {\ nu} ^ {\ sigma} $$, was gültig ist, weil $ \ partielle _ {\ mu} A _ {\ nu} $ $ d ^ 2 $ unabhängige Komponenten sind.

Wie kann man es tatsächlich beweisen?
Luboš Motl
2011-01-16 15:09:38 UTC
view on stackexchange narkive permalink

Lieber amc, schreibe zuerst deine Lagrange-Dichte als $$ L = - \ frac {1} {4} F _ {\ mu \ nu} F ^ {\ mu \ nu} = - \ frac {1} {2 } (\ partielle_ \ mu A_ \ nu) F ^ {\ mu \ nu} $$ Ist das soweit in Ordnung? Das $ F _ {\ mu \ nu} $ enthält zwei Terme, die es in den beiden Indizes antisymmetrisch machen. Es wird jedoch mit einem anderen $ F ^ {\ mu \ nu} $ multipliziert, das bereits antisymmetrisch ist, sodass ich es nicht erneut antisymmetrisieren muss. Stattdessen geben mir beide Terme dasselbe, sodass sich der Koeffizient $ -1 / 4 $ einfach in $ -1 / 2 $ ändert.

Nun zwingen Sie die Feldgleichungen, die Ableitungen des Lagrange zu berechnen in Bezug auf $ A_ \ mu $ und seine Derivate. Zunächst verschwindet die Ableitung der Lagrange-$ L $ in Bezug auf $ A_ \ mu $ -Komponenten selbst, da die Lagrange nur von den partiellen Ableitungen von $ A_ \ mu $ abhängt. Ist das bisher klar?

Die Bewegungsgleichungen lauten also $$ 0 = - \ Partial_ \ Mu [\ Partial L / \ Partial (\ Partial_ \ Mu A_ \ Nu)] = \ Dots $$ Hoppla, du bist schon an diesem Punkt angelangt. Aber jetzt schauen Sie sich meine Form des Lagrange oben an. Die Ableitung des Lagrange in Bezug auf $ \ partielle_ \ mu A_ \ nu $ ist einfach $$ - \ frac {1} {2} F ^ {\ mu \ nu} $$, weil $ \ partielle_ \ mu A_ \ nu $ erscheint einfach als Faktor, so dass die Bewegungsgleichungen einfach $$ 0 = + \ frac {1} {2} \ teilweise_ \ mu F ^ {\ mu \ nu} $$ sind. Ich habe jedoch absichtlich einen Fehler gemacht. Ich habe den Lagrange nur in Bezug auf $ \ partielle_ \ mu A_ \ nu $, die im ersten Faktor von $ F _ {\ mu \ nu} $ enthalten sind, mit den niedrigeren Indizes unterschieden. $ \ Partial_ \ mu A_ \ nu $ -Komponenten erscheinen jedoch auch in $ F ^ {\ mu \ nu} $, dem zweiten Faktor im Lagrange, einer mit den oberen Indizes. Wenn Sie die entsprechenden Begriffe aus der Leibniz-Regel hinzufügen, führt dies einfach dazu, dass sich der gesamte Beitrag verdoppelt. Die richtige Bewegungsgleichung, einschließlich des natürlichen Koeffizienten, lautet also $$ 0 = \ partielle_ \ mu F ^ {\ mu \ nu} $$ Die allgemeine Normalisierung ist wichtig, da diese Gleichung möglicherweise zusätzliche Terme wie den Strom erhält, dessen Koeffizient offensichtlich ist, und Sie keinen relativen Fehler von zwei zwischen der Ableitung von $ F $ und dem Strom $ j $ erhalten möchten.

CheersLubos

Hey, ich weiß, das ist 5 Jahre zu spät, aber vielleicht sehen Sie Folgendes: Warum ist $ \ frac {\ partiell} {\ partiell (\ partiell _ {\ mu} \ phi)} (\ partiell _ {\ mu} A _ {\ nu} F ^ {\ mu \ nu}) = F ^ {\ mu \ nu} $.Hängt der Tensor nicht auch von den partiellen Ableitungen ab?Müssen wir dann nicht die Produktregel verwenden?
Hallo @user17574 - hängt "welcher" Tensor nicht von den partiellen Ableitungen ab?Sicherlich der Spannungsenergietensor und der Lagrange.Deshalb ist die Ableitung davon in Bezug auf die partiellen Ableitungen ungleich Null.Die Ableitung wird in der Antwort berechnet.Die Produktregel funktioniert tatsächlich und deshalb storniert man den Faktor $ 1/2 $.Haben Sie versucht, die Antwort zu lesen?
Ich weiß, dass dies eine alte Frage ist, aber ich habe Zweifel.Nach Anwendung der Leibniz-Regel erhalten wir: $$ \ frac {\ partiell \ mathcal {L}} {\ partiell (\ partiell_ \ mu A_ \ nu)} = - \ frac {1} {2} \ left (\ partiell_ \ mu A_ \ nu \ frac {\partiell F ^ {\ mu \ nu}} {\ partiell (\ partiell_ \ mu A_ \ nu)} + \ frac {\ partiell_ \ mu A_ \ nu} {\ partiell (\ partiell_ \ mu A_ \ nu)} F ^{\ mu \ nu} \ right) $$ Wie soll ich das $ F ^ {\ mu \ nu} $ nach dem ersten Term der Gleichung unterscheiden?Ich verstehe nicht, wie ich dadurch $ F ^ {\ mu \ nu} $ bekomme.
Sie müssen ein neues Indexpaar verwenden, anstatt mu-nu dreimal zu kopieren.Dann ist $ \ partiell F ^ {\ alpha \ beta} / \ partiell (\ partiell_ \ gamma A_ \ delta) = g ^ {\ alpha \ gamma} g ^ {\ beta \ delta} - \ alpha \ leftrightarrow \ beta $ einfachweil $ F $ nur die Differenz zwischen zwei ähnlichen Begriffen ist (Antisymmetrisierung) und jeder Begriff die Ableitung hat, die im Grunde ein Kronecker-Delta ist - aber hier mit den erhöhten Indizes (um die Metrik des oberen Index zu werden) aufgrund der Position der Indizesim ursprünglichen Ausdruck.
Frobenius
2017-06-07 12:44:04 UTC
view on stackexchange narkive permalink

Obwohl ich spät in der Party bin, poste ich eine Antwort auf elementarer Ebene. Vielleicht beweist dies die Kraft der Tensorrechnung, die in allen vorherigen netten Antworten verwendet wurde.

Abstract

In dieser Antwort werden wir versuchen, Maxwell-Gleichungen im leeren Raum abzuleiten \ begin {align} \ boldsymbol {\ nabla} \ boldsymbol {\ times} \ mathbf {E} & = - \ frac {\ teilweise \ mathbf {B}} {\ teilweise t} \ tag {001a} \\ \ boldsymbol {\ nabla} \ boldsymbol {\ times} \ mathbf {B} & = \ mu_ {0} \ mathbf {j} + \ frac {1} {c ^ {2}} \ frac {\ teilweise \ mathbf { E}} {\ partielle t} \ tag {001b} \\ \ nabla \ boldsymbol {\ cdot} \ mathbf {E} & = \ frac {\ rho} {\ epsilon_ {0}} \ tag {001c} \\ \ nabla \ boldsymbol {\ cdot} \ mathbf {B} & = 0 \ tag {001d} \ end {align} aus den Euler-Lagrange-Gleichungen \ begin {Gleichung} \ boxed {\: \ dfrac {\ partiell} {\ partiell t} \ left (\ dfrac {\ partiell \ mathcal {L}} {\ partiell \ dot {\ eta} _ {\ jmath}} \ rechts) + \ nabla \ boldsymbol {\ cdot} \ left [\ dfrac {\ teilweise \ mathcal {L}} {\ teilweise \ links (\ boldsymbol {\ nabla} \ eta _ {\ jmath} \ rechts)} \ rechts] - \ frac {\ teilweise \ mathcal { L}} {\ partielle \ eta _ {\ jmath}} = 0, \ quad \ left (\ jmath = 1,2,3,4 \ right) \:} \ tag {002} \ end {Gleichung} wo \ begin {Gleichung} \ mathcal {L} = \ mathcal {L} \ left (\ eta _ {\ jmath}, \ dot {\ eta} _ {\ jmath}, \ boldsymbol {\ nabla} \ eta _ {\ jmath} \ right) \ qquad \ left (\ jmath = 1,2,3,4 \ right) \ tag {003} \ end {Gleichung} ist die Lagrange-Dichte der Frage (außer einem konstanten Faktor) \ begin {Gleichung} \ boxed {\: \ mathcal {L} = \ dfrac {\ Vert \ mathbf {E} \ Vert ^ {2} -c ^ {2} \ Vert \ mathbf {B} \ Vert ^ {2}} {2} + \ dfrac {1 } {\ epsilon_ {0}} \ left (- \ rho \ phi + \ mathbf {j} \ boldsymbol {\ cdot} \ mathbf {A} \ right) \:} \ tag {004} \ end {Gleichung} und $ \: \ eta _ {\ jmath} \ left (x_ {1}, x_ {2}, x_ {3}, t \ right), \: \: \ jmath = 1,2,3,4 \: $ die Komponenten $ \: A_ {1}, \: A_ {2}, \: A_ {3}, \ phi \: $ des EM-Potential-4-Vektors. In gewissem Sinne baut diese Ableitung auf der Umkehrung auf (dies ist das Finden einer richtigen Lagrange-Dichte aus Maxwell-Gleichungen), indem man sich rückwärts bewegt. Siehe meine Antwort hier: Deriving Lagrange-Dichte für elektromagnetisches Feld sup>

1. Hauptabschnitt

Zuerst drücken wir $ \: \ mathbf {E}, \ mathbf {B} \: $ von (004) als potenzielle 4-Vektor-Komponenten $ \: A_ {1}, \: A_ {2} aus , \: A_ {3}, \ phi \: $ \ begin {align} \ mathbf {B} & = \ boldsymbol {\ nabla} \ boldsymbol {\ times} \ mathbf {A} \ tag {005a} \\ \ mathbf {E} & = - \ boldsymbol {\ nabla} \ phi - \ dfrac {\ partiell \ mathbf {A}} {\ partiell t} = - \ boldsymbol {\ nabla} \ phi - \ mathbf {\ dot { EIN}} \ tag {005b} \ end {align} Ab (005) sind die Maxwell-Gleichungen (001a) und (001d) automatisch gültig. Daher müssen die vier (4) skalaren Maxwell-Gleichungen (001b) und (001c) aus den vier (4) skalaren Euler-Lagrange-Gleichungen (002) abgeleitet werden. Darüber hinaus ist anzunehmen, dass die Vektorgleichung (001b) aus (002) in Bezug auf die Komponenten des Vektorpotentials $ \: \ mathbf {A} = \ left (A_ {1}, \: A_ {) abgeleitet werden muss 2}, \: A_ {3} \ right) \: $, während die Skalargleichung (001c) aus (002) in Bezug auf das Skalarpotential $ \: \ phi \: $ abgeleitet werden muss.

Aus den Gleichungen (005) drücken wir die Lagrange-Dichte (004) als mögliche 4-Vektor-Komponenten $ \: A_ {1}, \: A_ {2}, \: A_ {3}, \ phi \ aus : $: \ begin {align} \ left \ Vert \ mathbf {E} \ right \ Vert ^ {2} & = \ left \ Vert - \ boldsymbol {\ nabla} \ phi - \ dfrac {\ partielle \ mathbf {A}} {\ partielle t} \ rechts \ Vert ^ {2} = \ links \ Vert \ mathbf {\ Punkt {A}} \ rechts \ Vert ^ {2} + \ Vert \ boldsymbol {\ nabla} \ phi \ Vert ^ {2} +2 \ links (\ boldsymbol {\ nabla} \ phi \ boldsymbol {\ cdot} \ mathbf {\ dot {A}} \ right) \ tag {006a} \\ & \keine Nummer\\ \ left \ Vert \ mathbf {B} \ right \ Vert ^ {2} & = \ left \ Vert \ boldsymbol {\ nabla} \ boldsymbol {\ times} \ mathbf {A} \ right \ Vert ^ {2} \ equiv \ sum ^ {k = 3} _ {k = 1} \ left [\ Vert \ boldsymbol {\ nabla} \ mathrm {A} _ {k} \ Vert ^ {2} - \ dfrac {\ teilweise \ mathbf {A. }} {\ partielle x_ {k}} \ boldsymbol {\ cdot} \ boldsymbol {\ nabla} \ mathrm {A} _ {k} \ right] \ tag {006b} \ end {align} Die zweite Gleichung in (006b) ist die Identität \ begin {Gleichung} \ left \ Vert \ boldsymbol {\ nabla} \ boldsymbol {\ times} \ mathbf {A} \ right \ Vert ^ {2} \ equiv \ sum ^ {k = 3} _ {k = 1} \ left [\ Vert \ boldsymbol {\ nabla} \ mathrm {A} _ {k} \ Vert ^ {2} - \ dfrac {\ partiell \ mathbf {A}} {\ partiell x_ {k}} \ boldsymbol {\ cdot} \ boldsymbol { \ nabla} \ mathrm {A} _ {k} \ right] \ tag {Id-01} \ end {Gleichung} ist in 2 bewiesen. Identities Section. Durch Einfügen von Ausdrücken (006) in (004) beträgt die Lagrange-Dichte \ begin {Gleichung} \ mathcal {L} = \ underbrace {\ tfrac {1} {2} \ left \ Vert \ mathbf {\ dot {A}} \ right \ Vert ^ {2} + \ tfrac {1} {2} \ Vert \ boldsymbol {\ nabla} \ phi \ Vert ^ {2} + \ boldsymbol {\ nabla} \ phi \ boldsymbol {\ cdot} \ mathbf {\ dot {A}}} _ {\ tfrac {1} {2} \ left \ Vert - \ boldsymbol {\ nabla} \ phi - \ frac {\ partiell \ mathbf {A}} {\ partiell t} \ rechts \ Vert ^ {2}} - \ tfrac {1} {2} c ^ {2 } \ underbrace {\ sum ^ {k = 3} _ {k = 1} \ left [\ Vert \ boldsymbol {\ nabla} \ mathrm {A} _ {k} \ Vert ^ {2} - \ frac {\ partiell \ mathbf {A}} {\ partielle x_ {k}} \ boldsymbol {\ cdot} \ boldsymbol {\ nabla} \ mathrm {A} _ {k} \ rechts]} _ {\ left \ Vert \ boldsymbol {\ nabla } \ boldsymbol {\ times} \ mathbf {A} \ right \ Vert ^ {2}} + \ frac {1} {\ epsilon_ {0}} \ left (- \ rho \ phi + \ mathbf {j} \ boldsymbol {\ cdot} \ mathbf {A} \ right) \ tag {007} \ end {Gleichung} sup>

Wir ordnen die Elemente in (007) wie folgt neu an:

\ begin {align} \ mathcal {L} & = \ overbrace {\ tfrac {1} {2} \ Vert \ boldsymbol {\ nabla} \ phi \ Vert ^ {2} - \ frac {\ rho \ phi} {\ epsilon_ {0}} + \ boldsymbol {\ nabla} \ phi \ boldsymbol {\ cdot} \ mathbf {\ dot {A}}} ^ {\ mathcal {L} _ {\ phi} = \ text {in Bezug auf} \ phi} + \ tfrac {1} {2} \ left \ Vert \ mathbf {\ dot {A}} \ right \ Vert ^ {2} + \ tfrac {1} {2} c ^ {2} \ sum ^ {k = 3} _ {k = 1} \ left [\ frac {\ partielle \ mathbf {A}} {\ partielle x_ {k}} \ boldsymbol {\ cdot} \ boldsymbol {\ nabla} \ mathrm {A} _ {k} - \ Vert \ boldsymbol {\ nabla} \ mathrm {A} _ {k} \ Vert ^ {2} \ right] + \ frac {\ mathbf {j} \ boldsymbol {\ cdot} \ mathbf {A}} {\ epsilon_ {0}} \ tag {008a} \\ \ mathcal {L} & = \ tfrac {1} {2} \ Vert \ boldsymbol {\ nabla} \ phi \ Vert ^ {2} - \ frac {\ rho \ phi} {\ epsilon_ {0}} + \ underbrace {\ boldsymbol {\ nabla} \ phi \ boldsymbol {\ cdot} \ mathbf {\ dot {A}} + \ tfrac {1} {2} \ left \ Vert \ mathbf {\ dot {A}} \ right \ Vert ^ {2} + \ tfrac {1} {2} c ^ {2} \ sum ^ {k = 3} _ {k = 1} \ left [\ frac {\ partielle \ mathbf {A}} {\ partielle x_ {k}} \ boldsymbol {\ cdot} \ boldsymbol {\ nabla} \ mathrm {A} _ {k} - \ Vert \ boldsymbol {\ nabla} \ mathrm {A} _ {k} \ Vert ^ {2} \ rechts] + \ frac {\ mathbf {j} \ boldsymbol {\ cdot} \ mathbf {A}} {\ epsilon_ {0}}} _ {\ mathcal {L} _ {\ mathbf {A}} = \ text { in Bezug auf} \ mathbf {A}} \ tag {008b} \ end {align} sup>

Der $ \: \ mathcal {L} _ {\ phi} \: $ -Teil der Dichte enthält alle $ \: \ phi $ -Terms und wird vernünftigerweise alleine an der Ableitung der Maxwell-Gleichung (001c) von teilnehmen die Euler-Lagrange-Gleichung (002) in Bezug auf $ \: \ eta_ {4} = \ phi \: $. Der Teil $ \: \ mathcal {L} _ {\ mathbf {A}} \: $ der Dichte enthält alle $ \: \ mathbf {A} $ - Terme und wird vernünftigerweise allein an der Ableitung der Maxwell-Gleichung teilnehmen ( 001b) aus den Euler-Lagrange-Gleichungen (002) in Bezug auf $ \: \ eta_ {1}, \ eta_ {2}, \ eta_ {3} = A_ {1}, A_ {1}, A_ {3} \ : $. Beachten Sie den allgemeinen Begriff $ \: \ boldsymbol {\ nabla} \ phi \ boldsymbol {\ cdot} \ mathbf {\ dot {A}} \: $ der Teile $ \: \ mathcal {L} _ {\ phi}, \ mathcal {L} _ {\ mathbf {A}} \: $.

Die Euler-Lagrange-Gleichung in Bezug auf $ \: \ eta_ {4} = \ phi \: $ lautet: \ begin {Gleichung} \ dfrac {\ partiell} {\ partiell t} \ overbrace {\ left (\ dfrac {\ partiell \ mathcal {L}} {\ partiell \ dot {\ phi}} \ rechts)} ^ {0} + \ nabla \ Boldsymbol {\ cdot} \ overbrace {\ left [\ dfrac {\ partiell \ mathcal {L}} {\ partiell \ links (\ boldsymbol {\ nabla} \ phi \ rechts)} \ rechts]} ^ {\ boldsymbol {\ nabla} \ phi + \ mathbf {\ dot {A}}} - \ overbrace {\ frac {\ partielle \ mathcal {L}} {\ partielle \ phi}} ^ {- \ frac {\ rho} {\ epsilon_ {0 }}} = 0 \ tag {009} \ end {Gleichung} oder \ begin {Gleichung} \ nabla \ boldsymbol {\ cdot} \ underbrace {\ left (- \ boldsymbol {\ nabla} \ phi - \ frac {\ partielle \ mathbf {A}} {\ partielle t} \ rechts)} _ {\ mathbf {E. }} = \ frac {\ rho} {\ epsilon_ {0}} \ tag {010} \ end {Gleichung} das ist die Maxwell-Gleichung (001c) \ begin {Gleichung} \ nabla \ boldsymbol {\ cdot} \ mathbf {E} = \ frac {\ rho} {\ epsilon_ {0}} \ tag {001c} \ end {Gleichung}

Um die Maxwell-Gleichung (001b) abzuleiten, drücken wir sie mit Hilfe der Gleichungen (005) in Form der möglichen 4-Vektor-Komponenten $ \: A_ {1}, \: A_ {2}, \: A_ aus. {3}, \ phi \: $: \ begin {Gleichung} \ boldsymbol {\ nabla} \ boldsymbol {\ times} \ left (\ boldsymbol {\ nabla} \ boldsymbol {\ times} \ mathbf {A} \ right) = \ mu_ {0} \ mathbf {j} + \ frac { 1} {c ^ {2}} \ frac {\ partiell} {\ partiell t} \ left (- \ boldsymbol {\ nabla} \ phi - \ frac {\ partiell \ mathbf {A}} {\ partiell t} \ Recht) \ tag {011} \ end {Gleichung} Die Identität benutzen \ begin {Gleichung} \ boldsymbol {\ nabla} \ boldsymbol {\ times} \ left (\ boldsymbol {\ nabla} \ boldsymbol {\ times} \ mathbf {A} \ right) = \ boldsymbol {\ nabla} \ left (\ nabla \ boldsymbol { \ cdot} \ mathbf {A} \ right) - \ nabla ^ {2} \ mathbf {A} \ tag {012} \ end {Gleichung} Gleichung (011) ergibt \ begin {Gleichung} \ frac {1} {c ^ {2}} \ frac {\ partiell ^ {2} \ mathbf {A}} {\ partiell t ^ {2}} - \ nabla ^ {2} \ mathbf {A} + \ Boldsymbol {\ nabla} \ left (\ nabla \ boldsymbol {\ cdot} \ mathbf {A} + \ frac {1} {c ^ {2}} \ frac {\ partiell \ phi} {\ partiell t} \ rechts) = \ mu_ {0} \ mathbf {j} \ tag {013} \ end {Gleichung} Die $ \: k $ -Komponente von Gleichung (013) wird richtig ausgedrückt, um wie eine Euler-Lagrange-Gleichung wie folgt auszusehen: \ begin {Gleichung} \ dfrac {\ partiell} {\ partiell t} \ links (\ frac {\ partiell \ mathrm {A} _ {k}} {\ partiell t} + \ frac {\ partiell \ phi} {\ partiell x_ {k} } \ rechts) + \ nabla \ boldsymbol {\ cdot} \ left [c ^ {2} \ left (\ frac {\ partielle \ mathbf {A}} {\ partielle x_ {k}} - \ boldsymbol {\ nabla} \ mathrm {A} _ {k} \ right) \ right] - \ frac {\ mathrm {j} _ {k}} {\ epsilon_ {0}} = 0 \ tag {014} \ end {Gleichung} Es reicht aus, über Gl. (014) aus der Euler-Lagrange-Gleichung (002) in Bezug auf $ \: \ eta_ {k} = A_ {k}, \: \: k = 1,2,3 \: $:

\ begin {Gleichung} \ dfrac {\ partiell} {\ partiell t} \ links (\ dfrac {\ partiell \ mathcal {L}} {\ partiell \ Punkt {A} _ {k}} \ rechts) + \ nabla \ boldsymbol {\ cdot} \ left [\ dfrac {\ partiell \ mathcal {L}} {\ partiell \ links (\ boldsymbol {\ nabla} A_ {k} \ rechts)} \ rechts] - \ frac {\ partiell \ mathcal {L}} { \ partielle A_ {k}} = 0 \ tag {015} \ end {Gleichung}

Jetzt \ begin {Gleichung} \ dfrac {\ partiell \ mathcal {L}} {\ partiell \ Punkt {A} _ {k}} = \ dfrac {\ partiell} {\ partiell \ Punkt {A} _ {k}} \ left (\ boldsymbol { \ nabla} \ phi \ boldsymbol {\ cdot} \ mathbf {\ dot {A}} + \ tfrac {1} {2} \ left \ Vert \ mathbf {\ dot {A}} \ right \ Vert ^ {2} \ right) = \ frac {\ partiell \ phi} {\ partiell x_ {k}} + \ frac {\ partiell \ mathrm {A} _ {k}} {\ partiell t} \ tag {016a} \ end {Gleichung}

\ begin {Gleichung} \ frac {\ partiell \ mathcal {L}} {\ partiell A_ {k}} = \ frac {\ partiell} {\ partiell A_ {k}} \ left (\ frac {\ mathbf {j} \ boldsymbol {\ cdot } \ mathbf {A}} {\ epsilon_ {0}} \ right) = \ frac {\ mathrm {j} _ {k}} {\ epsilon_ {0}} \ tag {016b} \ end {Gleichung} und \ begin {Gleichung} \ dfrac {\ partiell \ mathcal {L}} {\ partiell \ links (\ boldsymbol {\ nabla} A_ {k} \ rechts)} = \ dfrac {\ partiell} {\ partiell \ links (\ boldsymbol {\ nabla} A_ {k} \ rechts)} \ links (\ tfrac {1} {2} c ^ {2} \ sum ^ {k = 3} _ {k = 1} \ links [\ frac {\ partiell \ mathbf {A. }} {\ partielle x_ {k}} \ boldsymbol {\ cdot} \ boldsymbol {\ nabla} \ mathrm {A} _ {k} - \ Vert \ boldsymbol {\ nabla} \ mathrm {A} _ {k} \ Vert ^ {2} \ right] \ right) = c ^ {2} \ left (\ frac {\ partielle \ mathbf {A}} {\ partielle x_ {k}} - \ boldsymbol {\ nabla} \ mathrm {A. } _ {k} \ right) \ tag {016c} \ end {Gleichung} Die letzte Gleichung in (016c) ist aufgrund der in 2 nachgewiesenen Identität (Id-02) gültig. Identities Section: \ begin {Gleichung} \ dfrac {\ partiell \ links (\ links | \! \ links | \ boldsymbol {\ nabla} \ boldsymbol {\ times} \ mathbf {A} \ rechts | \! \ rechts | ^ {2} \ rechts)} { \ partiell \ links (\ boldsymbol {\ nabla} \ mathrm {A} _ {k} \ rechts)} = \ dfrac {\ partiell} {\ partiell \ links (\ boldsymbol {\ nabla} \ mathrm {A} _ { k} \ rechts)} \ links (\ sum ^ {k = 3} _ {k = 1} \ links [\ frac {\ partiell \ mathbf {A}} {\ partiell x_ {k}} \ boldsymbol {\ cdot } \ boldsymbol {\ nabla} \ mathrm {A} _ {k} - \ Vert \ boldsymbol {\ nabla} \ mathrm {A} _ {k} \ Vert ^ {2} \ right] \ right) = 2 \ left (\ boldsymbol {\ nabla} \ mathrm {A} _ {k} - \ frac {\ partielle \ mathbf {A}} {\ partielle x_ {k}} \ rechts) \ tag {Id-02} \ end {Gleichung} Unter Verwendung der Ausdrücke der Gleichungen (016) ergibt die Euler-Lagrange-Gleichung (015) (014) und damit die Maxwell-Gleichung (001b).

2. Identities Section

Wenn $ \: \ mathbf {A} = \ left (\ mathrm {A} _ {1}, \ mathrm {A} _ {2}, \ mathrm {A} _ {3} \ right) \: $ ist a Vektorfunktion der kartesischen Koordinaten $ \: \ left (x_ {1}, x_ {2}, x_ {3} \ right) \: $ then \ begin {Gleichung} \ left \ Vert \ boldsymbol {\ nabla} \ boldsymbol {\ times} \ mathbf {A} \ right \ Vert ^ {2} \ equiv \ sum ^ {k = 3} _ {k = 1} \ left [\ Vert \ boldsymbol {\ nabla} \ mathrm {A} _ {k} \ Vert ^ {2} - \ dfrac {\ partiell \ mathbf {A}} {\ partiell x_ {k}} \ boldsymbol {\ cdot} \ boldsymbol { \ nabla} \ mathrm {A} _ {k} \ right] \ tag {Id-01} \ end {Gleichung} und
\ begin {Gleichung} \ dfrac {\ partiell \ links (\ links | \! \ links | \ boldsymbol {\ nabla} \ boldsymbol {\ times} \ mathbf {A} \ rechts | \! \ rechts | ^ {2} \ rechts)} { \ partiell \ links (\ boldsymbol {\ nabla} \ mathrm {A} _ {k} \ rechts)} = \ dfrac {\ partiell} {\ partiell \ links (\ boldsymbol {\ nabla} \ mathrm {A} _ { k} \ rechts)} \ links (\ sum ^ {k = 3} _ {k = 1} \ links [\ frac {\ partiell \ mathbf {A}} {\ partiell x_ {k}} \ boldsymbol {\ cdot } \ boldsymbol {\ nabla} \ mathrm {A} _ {k} - \ Vert \ boldsymbol {\ nabla} \ mathrm {A} _ {k} \ Vert ^ {2} \ right] \ right) = 2 \ left (\ boldsymbol {\ nabla} \ mathrm {A} _ {k} - \ frac {\ partielle \ mathbf {A}} {\ partielle x_ {k}} \ rechts) \ tag {Id-02} \ end {Gleichung} wobei die funktionale Ableitung der linken Seite definiert ist als \ begin {Gleichung} \ dfrac {\ partiell \ links (\ links | \! \ links | \ boldsymbol {\ nabla} \ boldsymbol {\ times} \ mathbf {A} \ rechts | \! \ rechts | ^ {2} \ rechts)} { \ partiell \ links (\ boldsymbol {\ nabla} \ mathrm {A} _ {k} \ rechts)} \ equiv \ left [\ dfrac {\ partiell \ links (\ left | \! \ left | \ boldsymbol {\ nabla } \ boldsymbol {\ times} \ mathbf {A} \ right | \! \ right | ^ {2} \ right)} {\ partiell \ links (\ dfrac {\ partiell \ mathrm {A} _ {k}} { \ partielle x_ {1}} \ rechts)}, \ dfrac {\ partielle \ links (\ links | \! \ links | \ boldsymbol {\ nabla} \ boldsymbol {\ times} \ mathbf {A} \ rechts | \! \ rechts | ^ {2} \ rechts)} {\ partiell \ links (\ dfrac {\ partiell \ mathrm {A} _ {k}} {\ partiell x_ {2}} \ rechts)}, \ dfrac {\ partiell \ left (\ left | \! \ left | \ boldsymbol {\ nabla} \ boldsymbol {\ times} \ mathbf {A} \ right | \! \ right | ^ {2} \ right)} {\ partiell \ left ( \ dfrac {\ partielle \ mathrm {A} _ {k}} {\ partielle x_ {3}} \ rechts)} \ rechts] \ tag {Id-03} \ end {Gleichung} Beweis der Gleichung (Id-01): \ begin {eqnarray *} && \ left | \! \ Left | \ boldsymbol {\ nabla} \ boldsymbol {\ times} \ mathbf {A} \ right | \! \ right | ^ {2} = \ left (\ frac {\ partielle A_ {3}} {\ partielle x_ {2} } - \ frac {\ partielle A_ {2}} {\ partielle x_ {3}} \ rechts) ^ {2} + \ left (\ frac {\ partielle A_ {1}} {\ partielle x_ {3}} - \ frac {\ partielle A_ {3}} {\ partielle x_ {1}} \ rechts) ^ {2} + \ links (\ frac {\ partielle A_ {2}} {\ partielle x_ {1}} - \ frac {\ partielle A_ {1}} {\ partielle x_ {2}} \ rechts) ^ {2} \\ % ---------------------------------------- & = & \ left [\ left (\ frac {\ partielle A_ {1}} {\ partielle x_ {2}} \ rechts) ^ {2} + \ left (\ frac {\ partielle A_ {1}} {\ partiell x_ {3}} \ rechts) ^ {2} \ rechts] + \ links [\ links (\ frac {\ partiell A_ {2}} {\ partiell x_ {1}} \ rechts) ^ {2} + \ links (\ frac {\ partiell A_ {2}} {\ partiell x_ {3}} \ rechts) ^ {2} \ rechts] + \ links [\ links (\ frac {\ partiell A_ {3}} {\ partiell x_ {1}} \ rechts) ^ {2} + \ links (\ frac {\ partiell A_ {3}} {\ partiell x_ {2}} \ rechts) ^ {2} \ rechts] \\ % ---------------------------------------- &&-2 \ left [\ frac {\ partielle A_ {1}} {\ partielle x_ {2}} \ frac {\ partielle A_ {2}} {\ partielle x_ {1}} + \ frac {\ partielle A_ { 2}} {\ partielle x_ {3}} \ frac {\ partielle A_ {3}} {\ partielle x_ {2}} + \ frac {\ partielle A_ {3}} {\ partielle x_ {1}} \ frac {\ partielle A_ {1}} {\ partielle x_ {3}} \ rechts] \\ % ---------------------------------------- & = & \ left [\ left (\ frac {\ partielle A_ {1}} {\ partielle x_ {1}} \ rechts) ^ {2} + \ left (\ frac {\ partielle A_ {1}} {\ partiell x_ {2}} \ rechts) ^ {2} + \ links (\ frac {\ partiell A_ {1}} {\ partiell x_ {3}} \ rechts) ^ {2} \ rechts] + \ links [\ links (\ frac {\ partiell A_ {2}} {\ partiell x_ {1}} \ rechts) ^ {2} + \ links (\ frac {\ partiell A_ {2}} {\ partiell x_ {2}} \ rechts) ^ {2} + \ left (\ frac {\ partielle A_ {2}} {\ partielle x_ {3}} \ rechts) ^ {2} \ rechts] \\ % ---------------------------------------- && + \ left [\ left (\ frac {\ partielle A_ {3}} {\ partielle x_ {1}} \ rechts) ^ {2} + \ left (\ frac {\ partielle A_ {3}} {\ partielle x_ {2}} \ rechts) ^ {2} + \ links (\ frac {\ partiell A_ {3}} {\ partiell x_ {3}} \ rechts) ^ {2} \ rechts] - \ links [\ links ( \ frac {\ partielle A_ {1}} {\ partielle x_ {1}} \ rechts) ^ {2} + \ left (\ frac {\ partielle A_ {2}} {\ partielle x_ {2}} \ rechts) ^ {2} + \ left (\ frac {\ partiell A_ {3}} {\ partiell x_ {3}} \ rechts) ^ {2} \ rechts] \\ % ---------------------------------------- &&-2 \ left [\ frac {\ partielle A_ {1}} {\ partielle x_ {2}} \ frac {\ partielle A_ {2}} {\ partielle x_ {1}} + \ frac {\ partielle A_ { 2}} {\ partielle x_ {3}} \ frac {\ partielle A_ {3}} {\ partielle x_ {2}} + \ frac {\ partielle A_ {3}} {\ partielle x_ {1}} \ frac {\ partielle A_ {1}} {\ partielle x_ {3}} \ rechts] \\ % ---------------------------------------- & = & \ Vert \ boldsymbol {\ nabla} \ mathrm {A} _ {1} \ Vert ^ {2} + \ Vert \ boldsymbol {\ nabla} \ mathrm {A} _ {2} \ Vert ^ {2} + \ Vert \ boldsymbol {\ nabla} \ mathrm {A} _ {3} \ Vert ^ {2} - \ left (\ frac {\ partielle A_ {1}} {\ partielle x_ {1}} \ frac {\ partielles A_ {1}} {\ partielles x_ {1}} + \ frac {\ partielles A_ {2}} {\ partielles x_ {1}} \ frac {\ partielles A_ {1}} {\ partielles x_ {2} } + \ frac {\ partielle A_ {3}} {\ partielle x_ {1}} \ frac {\ partielle A_ {1}} {\ partielle x_ {3}} \ rechts) \\ % ---------------------------------------- &&- \ left (\ frac {\ partielle A_ {1}} {\ partielle x_ {2}} \ frac {\ partielle A_ {2}} {\ partielle x_ {1}} + \ frac {\ partielle A_ {2 }} {\ partielle x_ {2}} \ frac {\ partielle A_ {2}} {\ partielle x_ {2}} + \ frac {\ partielle A_ {3}} {\ partielle x_ {2}} \ frac { \ partielle A_ {2}} {\ partielle x_ {3}} \ rechts) - \ links (\ frac {\ partielle A_ {1}} {\ partielle x_ {3}} \ frac {\ partielle A_ {3}} {\ partielle x_ {1}} + \ frac {\ partielle A_ {2}} {\ partielle x_ {3}} \ frac {\ partielle A_ {3}} {\ partielle x_ {2}} + \ frac {\ partielles A_ {3}} {\ partielles x_ {3}} \ frac {\ partielles A_ {3}} {\ partielles x_ {3}} \ rechts) \\ % ---------------------------------------- & = & \ Vert \ boldsymbol {\ nabla} \ mathrm {A} _ {1} \ Vert ^ {2} + \ Vert \ boldsymbol {\ nabla} \ mathrm {A} _ {2} \ Vert ^ {2} + \ Vert \ boldsymbol {\ nabla} \ mathrm {A} _ {3} \ Vert ^ {2} - \ frac {\ partiell \ mathbf {A}} {\ partiell x_ {1}} \ boldsymbol {\ cdot} \ boldsymbol {\ nabla} \ mathrm {A} _ {1} - \ frac {\ partiell \ mathbf {A}} {\ partiell x_ {2}} \ boldsymbol {\ cdot} \ boldsymbol {\ nabla} \ mathrm { A} _ {2} - \ frac {\ teilweise \ mathbf {A}} {\ teilweise x_ {3}} \ boldsymbol {\ cdot} \ boldsymbol {\ nabla} \ mathrm {A} _ {3} \\ % ---------------------------------------- & = & \ sum ^ {k = 3} _ {k = 1} \ left [\ Vert \ boldsymbol {\ nabla} \ mathrm {A} _ {k} \ Vert ^ {2} - \ frac {\ partiell \ mathbf {A}} {\ partielle x_ {k}} \ boldsymbol {\ cdot} \ boldsymbol {\ nabla} \ mathrm {A} _ {k} \ right] \ end {eqnarray *} Beweis der Gleichung (Id-02): Aus der Gleichung \ begin {eqnarray *} && \ left | \! \ Left | \ boldsymbol {\ nabla} \ boldsymbol {\ times} \ mathbf {A} \ right | \! \ right | ^ {2} = \ left (\ frac {\ partielle A_ {3}} {\ partielle x_ {2} } - \ frac {\ partielle A_ {2}} {\ partielle x_ {3}} \ rechts) ^ {2} + \ left (\ frac {\ partielle A_ {1}} {\ partielle x_ {3}} - \ frac {\ partielle A_ {3}} {\ partielle x_ {1}} \ rechts) ^ {2} + \ links (\ frac {\ partielle A_ {2}} {\ partielle x_ {1}} - \ frac {\ partielle A_ {1}} {\ partielle x_ {2}} \ rechts) ^ {2} \\ % ---------------------------------------- & = & \ left [\ left (\ frac {\ partielle A_ {1}} {\ partielle x_ {2}} \ rechts) ^ {2} + \ left (\ frac {\ partielle A_ {1}} {\ partiell x_ {3}} \ rechts) ^ {2} \ rechts] + \ links [\ links (\ frac {\ partiell A_ {2}} {\ partiell x_ {1}} \ rechts) ^ {2} + \ links (\ frac {\ partiell A_ {2}} {\ partiell x_ {3}} \ rechts) ^ {2} \ rechts] + \ links [\ links (\ frac {\ partiell A_ {3}} {\ partiell x_ {1}} \ rechts) ^ {2} + \ links (\ frac {\ partiell A_ {3}} {\ partiell x_ {2}} \ rechts) ^ {2} \ rechts] \\ % ---------------------------------------- &&-2 \ left [\ frac {\ partielle A_ {1}} {\ partielle x_ {2}} \ frac {\ partielle A_ {2}} {\ partielle x_ {1}} + \ frac {\ partielle A_ { 2}} {\ partielle x_ {3}} \ frac {\ partielle A_ {3}} {\ partielle x_ {2}} + \ frac {\ partielle A_ {3}} {\ partielle x_ {1}} \ frac {\ partielle A_ {1}} {\ partielle x_ {3}} \ rechts] \ end {eqnarray *} wir haben \ begin {eqnarray *} \ dfrac {\ partiell \ links (\ links | \! \ links | \ boldsymbol {\ nabla} \ boldsymbol {\ times} \ mathbf {A} \ rechts | \! \ rechts | ^ {2} \ rechts)} { \ partiell \ links (\ dfrac {\ partiell \ mathrm {A} _ {1}} {\ partiell x_ {1}} \ rechts)} & = & 0 = 2 \ links (\ dfrac {\ partiell \ mathrm {A. } _ {1}} {\ partielle x_ {1}} - \ dfrac {\ partielle \ mathrm {A} _ {1}} {\ partielle x_ {1}} \ rechts) \\ \ dfrac {\ partiell \ links (\ links | \! \ links | \ boldsymbol {\ nabla} \ boldsymbol {\ times} \ mathbf {A} \ rechts | \! \ rechts | ^ {2} \ rechts)} { \ partiell \ links (\ dfrac {\ partiell \ mathrm {A} _ {1}} {\ partiell x_ {2}} \ rechts)} & = & 2 \ links (\ dfrac {\ partiell \ mathrm {A} _ {1}} {\ partielle x_ {2}} - \ dfrac {\ partielle \ mathrm {A} _ {2}} {\ partielle x_ {1}} \ rechts) \\ \ dfrac {\ partiell \ links (\ links | \! \ links | \ boldsymbol {\ nabla} \ boldsymbol {\ times} \ mathbf {A} \ rechts | \! \ rechts | ^ {2} \ rechts)} { \ partiell \ links (\ dfrac {\ partiell \ mathrm {A} _ {1}} {\ partiell x_ {3}} \ rechts)} & = & 2 \ links (\ dfrac {\ partiell \ mathrm {A} _ {1}} {\ partielle x_ {3}} - \ dfrac {\ partielle \ mathrm {A} _ {3}} {\ partielle x_ {1}} \ rechts) \ end {eqnarray *} Damit \ begin {Gleichung *} \ dfrac {\ partiell \ links (\ links | \! \ links | \ boldsymbol {\ nabla} \ boldsymbol {\ times} \ mathbf {A} \ rechts | \! \ rechts | ^ {2} \ rechts)} {\ partiell \ links (\ boldsymbol {\ nabla} \ mathrm {A} _ {1} \ rechts)} = 2 \ left (\ boldsymbol {\ nabla} \ mathrm {A} _ {1} - \ frac {\ partiell\ mathbf {A}} {\ teilweise x_ {1}} \ rechts) \ end {Gleichung *} Beweisgleichung (Id-02) für $ \: k = 1 \: $ und ähnlich für die beiden anderen Komponenten $ \: k = 2,3 $. sup>

Eric Angle
2014-01-03 07:43:22 UTC
view on stackexchange narkive permalink

Eine Methode besteht darin, die Maxwell-Aktion zu variieren (setzen Sie $ J ^ \ mu = 0 $, wenn Sie möchten, für den quellenfreien Fall) $$ S = \ int d ^ 4 x {\ mathcal {L}} = - \ int d ^ 4 x \ left (\ frac {1} {4} F ^ {\ mu \ nu} F _ {\ mu \ nu} + J ^ \ mu A_ \ mu \ right). $$ Beachten Sie zunächst, dass $$ \ begin {align} \ delta \ left (F ^ {\ mu \ nu} F _ {\ mu \ nu} \ right) & = 2 F ^ {\ mu \ nu} \ delta F _ {\ mu \ nu} \\ & = 2 F ^ {\ mu \ nu} \ left (\ Partial_ \ Mu \ Delta A_ \ Nu - \ Partial_ \ Nu \ Delta A_ \ Mu \ Right) \\ & = 4 F ^ {\ Mu \ Nu } \ Partial_ \ Mu \ Delta A_ \ Nu \\ & = 4 \ Left [\ Partial_ \ Mu \ Left (F ^ {\ Mu \ Nu} \ Delta A_ \ Nu \ Right) - \ Partial_ \ Mu F ^ {\ mu \ nu} \ delta A_ \ nu \ right], \ end {align} $$, wobei wir die Tatsache verwendet haben, dass $ F $ antisymmetrisch ist.

Beachten Sie auch, dass $ \ partielle_ \ mu \ left (F ^ {\ mu \ nu} \ delta A_ \ nu \ right) $ term verschwindet beim Konvertieren in ein Oberflächenintegral unter Verwendung des Standardarguments, dass $ \ delta A_ \ mu $ an der Integrationsgrenze verschwindet.

Unter Verwendung der obigen Angaben ist die Variation der Aktion $$ \ delta S = - \ int d ^ 4x \ \ delta A_ \ nu \ left (- \ p artial_ \ mu F ^ {\ mu \ nu} + J ^ \ nu \ right), $$, was, da $ \ delta A_ \ nu $ willkürlich ist, das gewünschte Ergebnis $$ \ partielle_ \ mu F ^ {\ mu ergibt \ nu} = J ^ \ nu. $$

Anstatt anzunehmen, dass $ \ delta A_ \ mu $ an der Grenze verschwindet, kann man $ F ^ {\ mu \ nu} $ an der Grenze annehmen.Ist das falsch?Eine verwandte Frage finden Sie hier https://physics.stackexchange.com/questions/438277/is-it-enough-to-assume-f-mu-nu-to-0-at-infinity-but-not-a-mu-zu-ableiten-t @EricAngle


Diese Fragen und Antworten wurden automatisch aus der englischen Sprache übersetzt.Der ursprüngliche Inhalt ist auf stackexchange verfügbar. Wir danken ihm für die cc by-sa 2.0-Lizenz, unter der er vertrieben wird.
Loading...